Question and Answers Forum

All Questions      Topic List

Others Questions

Previous in All Question      Next in All Question      

Previous in Others      Next in Others      

Question Number 20316 by Tinkutara last updated on 25/Aug/17

If at a height of 40 m, the direction of  motion of a projectile makes an angle  π/4 with the horizontal, then its initial  velocity and angle of projection are,  respectively  (a) 30, (1/2)cos^(−1) (−(4/5))  (b) 30, (1/2)cos^(−1) (−(1/2))  (c) 50, (1/2)cos^(−1) (−(8/(25)))  (d) 60, (1/2)cos^(−1) (−(1/4))

$$\mathrm{If}\:\mathrm{at}\:\mathrm{a}\:\mathrm{height}\:\mathrm{of}\:\mathrm{40}\:\mathrm{m},\:\mathrm{the}\:\mathrm{direction}\:\mathrm{of} \\ $$$$\mathrm{motion}\:\mathrm{of}\:\mathrm{a}\:\mathrm{projectile}\:\mathrm{makes}\:\mathrm{an}\:\mathrm{angle} \\ $$$$\pi/\mathrm{4}\:\mathrm{with}\:\mathrm{the}\:\mathrm{horizontal},\:\mathrm{then}\:\mathrm{its}\:\mathrm{initial} \\ $$$$\mathrm{velocity}\:\mathrm{and}\:\mathrm{angle}\:\mathrm{of}\:\mathrm{projection}\:\mathrm{are}, \\ $$$$\mathrm{respectively} \\ $$$$\left({a}\right)\:\mathrm{30},\:\frac{\mathrm{1}}{\mathrm{2}}\mathrm{cos}^{−\mathrm{1}} \left(−\frac{\mathrm{4}}{\mathrm{5}}\right) \\ $$$$\left({b}\right)\:\mathrm{30},\:\frac{\mathrm{1}}{\mathrm{2}}\mathrm{cos}^{−\mathrm{1}} \left(−\frac{\mathrm{1}}{\mathrm{2}}\right) \\ $$$$\left({c}\right)\:\mathrm{50},\:\frac{\mathrm{1}}{\mathrm{2}}\mathrm{cos}^{−\mathrm{1}} \left(−\frac{\mathrm{8}}{\mathrm{25}}\right) \\ $$$$\left({d}\right)\:\mathrm{60},\:\frac{\mathrm{1}}{\mathrm{2}}\mathrm{cos}^{−\mathrm{1}} \left(−\frac{\mathrm{1}}{\mathrm{4}}\right) \\ $$

Commented by Tinkutara last updated on 25/Aug/17

I think velocity of projection should be  given. Isn′t it?

$$\mathrm{I}\:\mathrm{think}\:\mathrm{velocity}\:\mathrm{of}\:\mathrm{projection}\:\mathrm{should}\:\mathrm{be} \\ $$$$\mathrm{given}.\:\mathrm{Isn}'\mathrm{t}\:\mathrm{it}? \\ $$

Commented by ajfour last updated on 25/Aug/17

one more information need be  given..

$${one}\:{more}\:{information}\:{need}\:{be} \\ $$$${given}.. \\ $$

Commented by Tinkutara last updated on 25/Aug/17

This is the wrong solution given in  book or it is correct?

$$\mathrm{This}\:\mathrm{is}\:\mathrm{the}\:\mathrm{wrong}\:\mathrm{solution}\:\mathrm{given}\:\mathrm{in} \\ $$$$\mathrm{book}\:\mathrm{or}\:\mathrm{it}\:\mathrm{is}\:\mathrm{correct}? \\ $$

Commented by Tinkutara last updated on 25/Aug/17

Commented by Joel577 last updated on 25/Aug/17

how did it say 50 m/s?

$${how}\:{did}\:{it}\:{say}\:\mathrm{50}\:{m}/{s}? \\ $$

Commented by Tinkutara last updated on 25/Aug/17

Yes, so this solution is wrong but it is  in my book.

$$\mathrm{Yes},\:\mathrm{so}\:\mathrm{this}\:\mathrm{solution}\:\mathrm{is}\:\mathrm{wrong}\:\mathrm{but}\:\mathrm{it}\:\mathrm{is} \\ $$$$\mathrm{in}\:\mathrm{my}\:\mathrm{book}. \\ $$

Commented by ajfour last updated on 25/Aug/17

−1≤ cos 2θ ≤1  −1 ≤ −((2gh)/u^2 ) ≤ 1  ⇒ u^2  ≥ 2gh       u^2  ≥ 800      u ≥ 20(√2)    (≈ 28m/s ).  if  u=30m/s ⇒  θ=(1/2)cos^(−1) (−(8/9))  if u=40m/s ⇒  θ=(1/2)cos^(−1) (−(1/2))  if u=50m/s ⇒  θ=(1/2)cos^(−1) (−(8/(25)))  if u=60m/s ⇒ θ=(1/2)cos^(−1) (−(2/9))  (c) is one of the possible solutions  (a), (b), (d) are just not possible.

$$−\mathrm{1}\leqslant\:\mathrm{cos}\:\mathrm{2}\theta\:\leqslant\mathrm{1} \\ $$$$−\mathrm{1}\:\leqslant\:−\frac{\mathrm{2}{gh}}{{u}^{\mathrm{2}} }\:\leqslant\:\mathrm{1} \\ $$$$\Rightarrow\:{u}^{\mathrm{2}} \:\geqslant\:\mathrm{2}{gh}\:\: \\ $$$$\:\:\:{u}^{\mathrm{2}} \:\geqslant\:\mathrm{800} \\ $$$$\:\:\:\:{u}\:\geqslant\:\mathrm{20}\sqrt{\mathrm{2}}\:\:\:\:\left(\approx\:\mathrm{28}{m}/{s}\:\right). \\ $$$${if}\:\:{u}=\mathrm{30}{m}/{s}\:\Rightarrow\:\:\theta=\frac{\mathrm{1}}{\mathrm{2}}\mathrm{cos}^{−\mathrm{1}} \left(−\frac{\mathrm{8}}{\mathrm{9}}\right) \\ $$$${if}\:{u}=\mathrm{40}{m}/{s}\:\Rightarrow\:\:\theta=\frac{\mathrm{1}}{\mathrm{2}}\mathrm{cos}^{−\mathrm{1}} \left(−\frac{\mathrm{1}}{\mathrm{2}}\right) \\ $$$${if}\:{u}=\mathrm{50}{m}/{s}\:\Rightarrow\:\:\theta=\frac{\mathrm{1}}{\mathrm{2}}\mathrm{cos}^{−\mathrm{1}} \left(−\frac{\mathrm{8}}{\mathrm{25}}\right) \\ $$$${if}\:{u}=\mathrm{60}{m}/{s}\:\Rightarrow\:\theta=\frac{\mathrm{1}}{\mathrm{2}}\mathrm{cos}^{−\mathrm{1}} \left(−\frac{\mathrm{2}}{\mathrm{9}}\right) \\ $$$$\left({c}\right)\:{is}\:{one}\:{of}\:{the}\:{possible}\:{solutions} \\ $$$$\left({a}\right),\:\left({b}\right),\:\left({d}\right)\:{are}\:{just}\:{not}\:{possible}. \\ $$

Commented by Tinkutara last updated on 25/Aug/17

Thanks you very much Sir! So it is to  be seen by the options.

$$\mathrm{Thanks}\:\mathrm{you}\:\mathrm{very}\:\mathrm{much}\:\mathrm{Sir}!\:\mathrm{So}\:\mathrm{it}\:\mathrm{is}\:\mathrm{to} \\ $$$$\mathrm{be}\:\mathrm{seen}\:\mathrm{by}\:\mathrm{the}\:\mathrm{options}. \\ $$

Terms of Service

Privacy Policy

Contact: info@tinkutara.com